9. a) A computer can finish to download an application file in 4 minutes of 600 MB per minute, 6.1 P
(1) Find the size of the application file. 1. L
(ii) How long does it take to download the file when the download rate increase
to 800 MB per minutes?​

Answers

Answer 1

Answer:

1) 2400MB

2) 3 minutes

Step-by-step explanation:

1) 600 MB/Min * 4 Min = 2400MB

2) at 800 MB/Min a 2400MB file will require 2400MB/800MB/Min

  2400/800 = 3Min


Related Questions

1, 1, 1, 1, 1, 1, 2, 3, 5, 8, 13, 21, 34, 55, 89, 89, 89, 89, 89, 89

At what percentile is 34? ​

Answers

Answer:

6 out of the 20 values are below 2.  6/20 = 30%.

Since 30% of the values are below 2, 2 is located at the 30th percentile.

Step-by-step explanation:

ayudenme con esta ecuacion de igualacion ¿ resuelve cada sistema de ecuacion por el metodo de igualacion?
7x + 4y = 2
x + y= 1

Answers

Answer:

[tex]{x, y} = {-\frac{2}{3}, \frac{5}{3}}[/tex]

Step-by-step explanation:

7x + 4y = 2

x + y = 1

// Solve equation [2] for the variable  y  

 

 [2]    y = -x + 1

// Plug this in for variable  y  in equation [1]

  [1]    7x + 4•(-x +1) = 2

  [1]    3x = -2

// Solve equation [1] for the variable  x  

  [1]    3x = - 2  

  [1]    x = - 2/3  

// By now we know this much :

   x = -2/3

   y = -x+1

// Use the  x  value to solve for  y  

   y = -(-2/3)+1 = 5/3  

Solution :

{x,y} = {-2/3,5/3}

Maths!
1) Calculate the variance and standard division of the set of the data

2) If each value is added by 2, calculate the new standard deviation of the set

3) What is the effect on the measure of dispersion if each value is changed uniformly ​

Answers

Answer:

(1) Variance = 4.5 and Standard deviation = 2.121.

(2) Variance = 4.5 and Standard deviation = 2.121.

(3) The effect on the measure of dispersion if each value is changed uniformly ​is that it remains unchanged.

Step-by-step explanation:

We are given with the following set of data below;

              X                     [tex]X-\bar X[/tex]                   [tex](X-\bar X)^{2}[/tex]

              5                   5 - 8 = -3                     9

              5                   5 - 8 = -3                     9

              8                   8 - 8 = 0                      0

             10                  10 - 8 = 2                     4

             10                  10 - 8 = 2                     4

             10                  10 - 8 = 2                     4

              9                   9 - 8 = 1                       1

              9                   9 - 8 = 1                       1

              6                  6 - 8 = -2                    4                

Total    72                                                   36      

Firstly, the mean of the above data is given by;

              Mean, [tex]\bar X[/tex]  =  [tex]\frac{\sum X}{n}[/tex]

                              =  [tex]\frac{72}{9}[/tex]  = 8

(1)Now, the variance of the given data is;

            Variance  =  [tex]\frac{\sum (X-\bar X)^{2} }{n-1}[/tex]

                                  =  [tex]\frac{36}{9-1}[/tex]  = 4.5

So, the standard deviation, (S.D.)  =  [tex]\sqrt{\text{Variance}}[/tex]

                                                        =  [tex]\sqrt{4.5}[/tex] = 2.12

(2) Now, each value is added by 2; so the new data set is given by;

              X                     [tex]X-\bar X[/tex]                   [tex](X-\bar X)^{2}[/tex]

              7                   7 - 10 = -3                     9

              7                   7 - 10 = -3                     9

             10                  10 - 10 = 0                     0

             12                  12 - 10 = 2                     4

             12                  12 - 10 = 2                     4

             12                  12 - 10 = 2                     4

              11                  11 - 10 = 1                       1

              11                  11 - 10 = 1                       1

              8                  8 - 10 = -2                    4                

Total    90                                                   36      

Firstly, the mean of the above data is given by;

              Mean, [tex]\bar X[/tex]  =  [tex]\frac{\sum X}{n}[/tex]

                              =  [tex]\frac{90}{9}[/tex]  = 10

(1)Now, the variance of the given data is;

            Variance  =  [tex]\frac{\sum (X-\bar X)^{2} }{n-1}[/tex]

                                  =  [tex]\frac{36}{9-1}[/tex]  = 4.5

So, the new standard deviation, (S.D.)  =  [tex]\sqrt{\text{Variance}}[/tex]

                                                                =  [tex]\sqrt{4.5}[/tex] = 2.12

(3) The effect on the measure of dispersion if each value is changed uniformly ​is that it remains unchanged as we see in the case of variance or standard deviation.

3(-2-4v)<-90 help I'm stuck on this one

Answers

Well, if you disregard the signs for a second, you can work this like a normal equation:
-6 - 12v = 90
-12v = 96
v = 8
Now just reincorporate the less than sign:
v < 8

How do I solve this?​

Answers

Answer:

[tex]\Large \boxed{- \frac{1}{5}}[/tex]

Step-by-step explanation:

[tex]\displaystyle \sqrt[3]{-\frac{1}{125} }[/tex]

Distribute the cube root to the numerator and denominator.

[tex]\displaystyle -\frac{\sqrt[3]{1} }{\sqrt[3]{125} }[/tex]

Solve for the cube root.

[tex]\displaystyle - \frac{1}{5}[/tex]

HELP ASAP WILL MARK BRAINLIEST!!!!!! Use the number line​ below, where ​RS=9y+2, ST = 4y+9 and RT = 115. a. What is the value of​ y? b. Find RS and ST. a. What is the value of​ y?

Answers

Answer: y = 7

Step-by-step explanation:

HELP ME PLEASEEEE

TAXI RATES A New York City taxi charges $2.50, plus $.40 for each fifth of a
mile if it is not delayed by traffic. Write an expression for the cost of the ride
if you travel x miles in the taxi with no traffic delays.

Answers

Step-by-step explanation:

C(x) = 2.50 + 5(0.40x)

<=> C(x) = 2.50 + 2.00 x

The required expression for the taxi charges is C(x) = $2.50 + 0.4  * x/5.

Given that,
A New York City taxi charges $2.50, plus $.40 for each fifth of a
mile if it is not delayed by traffic. To determine an expression for the cost of the ride, if travel x miles in the taxi with no traffic delays to determined.

What is arithmetic?

In mathematics, it deals with numbers of operations according to the statements. There are four major arithmetic operators, addition, subtraction, multiplication and division,

Here,
Let the number of miles be x,
The total charge be C(x)
Now accoding to the condition total charge could be the sum of the cost of every fifth mile and $2.40. So, every fifth mile = x / 5
Now,
C(x) = 2.40 + 0.40 * x/5

Thus, the required expression for the taxi charges is C(x) = $2.50 + 0.4  * x/5.

Learn more about arithmetic here:

brainly.com/question/14753192

#SPJ2


Determine if the following set of ordered pairs represents a quadratic function. Explain.
(5, 7), (7, 11). (9, 14). (11, 18)
(1 point)​

Answers

Answer:

Option D

Step-by-step explanation:

Given

[tex](5, 7), (7, 11). (9, 14). (11, 18)[/tex]

Required

Determine if the function is quadratic

[tex](5, 7), (7, 11). (9, 14). (11, 18)[/tex]

Start by representing the function as an x-y table

x:-  5 || 7 || 9 || 11

y:-  7 ||  11 || 14 || 18

Next; Calculate the difference between the values of y

[tex]Difference: 11 - 7 = 4[/tex]

[tex]Difference: 14 - 11 = 3[/tex]

[tex]Difference: 18 - 4 = 4[/tex]

The resulting difference are: 4 || 3 || 4

Next; Calculate the difference between the difference of values of y

[tex]Difference = 3 - 4 = -1[/tex]

[tex]Difference = 4 - 3 = 1[/tex]

The resulting difference are: -1 || 1

For the function to be quadratic, the above difference must be the same and since they are not, then the function does not represent a quadratic function.

Option D answers the question

Based on the graph what are the solutions to ax^2 + bx+c=0 Select all that apply

A. X=-2

B. X=10

C. X=5

D. X=8

Answers

Answer:

x=-2 and x = 5

Step-by-step explanation:

The solutions to the equation are where the graph crosses the x axis

We can see that the graph crosses at x=-2 and x = 5

Answer:

x = -2

x = 5

Step-by-step explanation:

The answer is found when the graph crosses the x-axis

Hope this helps :D

Celine is Drake’s granddaughter. Her age is 4 years greater than of Drake’s age. If Celine is 28 years old, how old is Drake?

Answers

Answer:32

Step-by-step explanation:

if selling is 28 and she is 4 years greater than Drake then that is 28-4 which is 32 so Drake is 32 years old

Answer:

The answer is 32.

Step-by-step explanation:

If Celine is 28 and Drake is four years older than her, we do 28+4.


What is the surface area of the cylinder?
180pi ft2
192pi ft2
252pi ft2
396pi ft2

Answers

[tex]\sf \longrightarrow \: \:Surface \: area \: of \: cylinder \: = \: 2\pi \: r \: h \: + \: 2\pi \: {r}^{2} [/tex]

r represents radius of cylinder

h represents height of cylinder

[tex]\sf \longrightarrow \: \: r \: = \: \frac{Diameter}{2} \\ [/tex]

[tex]\sf \longrightarrow \: \: r \: = \: \frac{12}{2} \\ [/tex]

[tex]\sf \longrightarrow \: \: r \: = \: \cancel\frac{12}{2} \: \: ^{6} \\ [/tex]

[tex]\sf \longrightarrow \: \: r \: = \:6 \: ft[/tex]

[tex]\sf \longrightarrow \: \: h \: = \: 15 \: ft[/tex]

Now , Substuting the values

[tex]\sf \implies \: \:2 \: \times \: \pi \times \: 6 \: \times \: 15 \: \: + \: \: 2 \: \times \: \pi \times \: {6}^{2} [/tex]

[tex]\sf \implies \: \:2 \: \times \: \pi \times \: 90 \: \: + \: \: 2 \: \times \: \pi \times \: 36[/tex]

[tex]\sf \implies \: \:180 \: \pi \: + \: \: 72 \: \pi[/tex]

[tex]\sf \implies \: \:252 \: \pi[/tex]

Hence, the surface area of cylinder is 252 π ft²

Find the value of the expression x3−y3+x2+y2+3 by substituting x=−2 and y=1.

Answers

[tex]\\ \sf\dashrightarrow x^3-y^3+x^2+y^2+3[/tex]

[tex]\\ \sf\dashrightarrow (-2)^3-(1)^3+(-2)^2+(1)^2+3[/tex]

[tex]\\ \sf\dashrightarrow -8-1+4+1+3[/tex]

[tex]\\ \sf\dashrightarrow -9+8[/tex]

[tex]\\ \sf\dashrightarrow -1[/tex]

Answer: -1

Concept:

Here, we need to understand the idea of evaluation.  

When encountering questions that gave you an expression with variables, then stated: "If x = a, y = b, z = c" (a, b, c are all constants), this means you should substitute the value given for each variable back to the expression.

Solve:

Given information

x = -2

y = 1

Given expression

x³ - y³ + x² + y² + 3

Substitute values into the expression

=(-2)³ - (1)³ + (-2)² + (1)² + 3

Simplify exponents

=-8 - 1 + 4 + 1 + 3

Simplify by simple operation (addition/subtraction)

=-9 + 4 + 1 + 3

=-5 + 1 + 3

=-4 + 3

=[tex]\boxed {-1}[/tex]

Hope this helps!! :)

Please let me know if you have any questions

Solve for x(in picture).

Answers

Answer:

x = 1/8

Step-by-step explanation:

[tex]\log _2\left(x\right)=-3\\\\\log _a\left(b\right)=c\:\mathrm{then}\:b=a^c\\\\\log _2\left(x\right)=-3\quad \Rightarrow \quad \:x=2^{-3}\\\\Simplify\\\\x=\frac{1}{8}[/tex]

write brackets () in this statement to make it correct 6 + 4 x 5 - 3 = 14

Answers

Answer:

6+4*(5-3)

Step-by-step explanation:

Because of PEMDAS l, we first solve in the parantheses. So 5-3 would be 2, then we multiply 2 by 4 which would be 8 obviously. Then we do the final step, add 6 and 8 which would be 14. Cheers!

Answer: 6 + 4 × (5-3)

Explanation:

6 + 4 × (5-3) = 14

6 + 4 × 2 = 14

6 + 8 = 14

14 = 14

Must click thanks and mark brainliest

Determine whether or not the given pair of triangles is similar and state how you know.
Is ΔGHI ~ ΔJKI?
Mark all that apply.

Answers

Answer:

            by SAS.                Yes  

Step-by-step explanation:

∠HIG and ∠JIK are vertical angles, so:

m∠HIG = m∠JIK

[tex]\dfrac{GI}{IJ}=\dfrac{7.5}3=2.5\\\\\dfrac{HI}{IK}=\dfrac{2.5}1=2.5[/tex]

[tex]\left\{\dfrac{GI}{IJ}=\dfrac{HI}{IK}\quad\ \wedge\quad m\angle HIG=m\angle JIK\right\}\implies \triangle HIG\sim\triangle JIK\ \ by\ SAS[/tex]

Guys please help me find the answer to this

Answers

Answer:

C

Step-by-step explanation:

The colony will follow an exponential equation. y(x) = 8000*(2)^(t/210). Plug in t=630 and you will get y(x) = 64000

PLEASE HELP QWQ AsAp with these 4 questions

Answers

Answer:

Step-by-step explanation:

I can't believe I'm doing this for 5 points, but ok!

For the first 3, we are going to multiply to find the value of that 3 x 3 matrix by picking up the first 2 columns and plopping them down at the end and then multiplying through using the rules for multiplying matrices:

[tex]\left[\begin{array}{ccccc}7&4&6&7&4\\-4&8&9&-4&8\\1&8&7&1&8\end{array}\right][/tex]  and from there find the sum of the products of the main axes minus the sum of the products of the minor axes, as follows (I'm not going to state the process in the next 2 problems, so make sure you follow it here. This is called the determinate. The determinate is what you get when you evaluate or find the value of a matrix. Just so you know):

[tex](7*8*7)+(4*9*1)+(6*-4*8)-[(1*8*6)+(8*9*7)+(7*-4*4)][/tex] which gives us:

392 + 36 - 192 - [48 + 504 - 112] which simplifies to

236 - 440 which is -204

On to the second one:

[tex]\left[\begin{array}{ccccc}-8&-4&-1&-8&-4\\1&7&-3&1&7\\8&9&9&8&9\end{array}\right][/tex] and multiplying gives us

[tex](-8*7*9)+(-4*-3*8)+(-1*1*9)-[(8*7*-1)+(9*-3*-8)+(9*1*-4)][/tex] which gives us:

-504 + 96 - 9 - [-56 + 216 - 36] which simplifies to

-417 - 124 which is -541, choice c.

Now for the third one:

[tex]\left[\begin{array}{ccccc}-2&-2&-5&-2&-2\\2&7&-3&2&7\\8&9&9&8&9\end{array}\right][/tex] and multiplying gives us

[tex](-2*7*9)+(-2*-3*8)+(-5*2*9)-[(8*7*-5)+(9*-3*-2)+(9*2*-2)][/tex] which gives us:

[tex]-126+48-90-[-280+54-36][/tex] which simplifies to

-168 - (-262) which is 94, choice c again.

Now for the last one. I'll show you the set up for the matrix equation; I solved it using the inverse matrix. So I'll also show you the inverse and how I found it.

[tex]\left[\begin{array}{cc}-4&-5&\\-6&-8\\\end{array}\right][/tex] [tex]\left[\begin{array}{c}x\\y\\\end{array}\right][/tex] = [tex]\left[\begin{array}{c}-5\\-2\\\end{array}\right][/tex] and I found the inverse of the 2 x 2 matrix on the left.

Find the inverse by:

* finding the determinate

* putting the determinate under a 1

* multiply that by the "mixed up matrix (you'll see...)

First things first, the determinate:

|A| = (-4*-8) - (-6*-5) which simplifies to

|A| = 32 - 30 so

|A| = 2; now put that under a 1 and multiply it by the mixed up matrix. The mixed up matrix is shown in the next step:

[tex]\frac{1}{2}\left[\begin{array}{cc}-8&5\\6&-4\end{array}\right][/tex]  (to get the mixed up matrix, swap the positions of the numbers on the main axis and then change the signs of the numbers on the minor axis). Now we multiply in the 1/2 to get the inverse:

[tex]\left[\begin{array}{cc}-4&\frac{5}{2}\\3&-2\\\end{array}\right][/tex] Multiply that inverse by both sides of the equation. This inverse "undoes" the matrix that's already there (like dividing the matrix that's already there by itself) which leaves us with just the matrix of x and y. Multiply the inverse matrix by the solution matrix:

[tex]\left[\begin{array}{c}x&y\end{array}\right] =\left[\begin{array}{cc}-4&\frac{5}{2} \\3&-2\end{array}\right] *\left[\begin{array}{c}-5&-2\\\end{array}\right][/tex] and that right side multiplies out to

x = 20 - 5 which is

x = 15 and

y = -15 + 4 which is

y = -11

(It works, I checked it)

Find the interest rate if $8200 has
a final value of $11406 in 4 years.
Give your answer to 1 d.p.

Answers

r=I × 100÷pxt

8200×100÷ 11406×4

820000÷. 45624

17.97

make me brainliest .

Correct answer gets brainliest and 5 stars

Answers

Answer:

5, 12, 13

Step-by-step explanation:

Pythagoras theorem ⇒ Perpendicular² + Base² = Hypotenuse²

5² + 12² = 13²

25 + 144 = 169

Thus 5, 12, 13 is a Pythagorean triple.

Can you help Jorge organize the results into a two-way frequency table?

Answers

Answer:

Top left: 6

Bottom left: 8

Top right: 7

Bottom right: 3

Step-by-step explanation:

To solve this problem, we need to look at the table to see what missing information we need to solve for. On the table, it asks us how many students:

Play a musical instrument and sportDoesn't play anythingPlays sportsPlays an instrument

On the first intersection (top left), we are asked how many students played an instrument and sport. That is easy to solve because the information they provide us already gives us that number: 6.

On the second intersection (bottom left), things get a little more challenging. The information states that 14 students in total play sports, so that is not the number of people who only play sports. Since 14 people in total play sports and 6 people play both an instrument and sport, we subtract 6 from 14 to get our answer, 8.

On the third intersection (top right), it asks us how many people play an instrument but doesn't play a sport. Let's look at the remaining values first. There is: 6, 8, 3. When we add those together, we get 17, and when we subtract that from the total (24), we get our answer: 7.

On the last intersection (bottom right), the information already provides us with the answer to how many people don't play anything: 3.

please help me i offered all my points and this is really important!!! The question is attached.

Answers

Answer:

25[tex]\sqrt{3}[/tex] +60

Step-by-step explanation: The first thing you need to do is realize that, this figure is a isosceles trapezoid due to the markings on each side.

So now we know both sides are 10.

We also know the the top two angles are congruent to each other and so are the bottom two angles due to the trapezoid being isosceles.

So the top two angles are 120 degrees and bottom two angles are 60 degrees.

It seems like we can't find the sides, let's try drawing two lines from each top angle all the way down to form two right triangles.

Wow, these two triangles are special right triangles in the form of

30 - 60 - 90 degrees.

shorter side = n

longer side = n[tex]\sqrt{3}[/tex]

hypotenuse = 2n

So, 2n = 10

n = 5 for the short side

The bottom base is 4[tex]\sqrt{3}[/tex] + 5 + 5 = 10 + 4[tex]\sqrt{3}[/tex]

The longer side is  5[tex]\sqrt{3}[/tex].

The area of trapezoid = (base1 + base2)/2 * height

= (4[tex]\sqrt{3}[/tex] + 10 + 4[tex]\sqrt{3}[/tex])/2 * 5[tex]\sqrt{3}[/tex] = (10 + 8[tex]\sqrt{3}[/tex])/2 * 5[tex]\sqrt{3}[/tex] = (5+4[tex]\sqrt{3}[/tex])*5[tex]\sqrt{3}[/tex] = 25[tex]\sqrt{3}[/tex] +60

So, 25[tex]\sqrt{3}[/tex] + 60 is our answer.

Answer:

  60 +25√3

Step-by-step explanation:

In the figure of the isosceles trapezoid below, the angles at C and D are supplementary to the given angle, so are 60°. That makes triangle BDE a 30°-60°-90° right triangle, which has side length ratios ...

  DE : BE : BD = 1 : √3 : 2 = 5 : 5√3 : 10

Triangle BDE can be relocated to the other end of the figure to become triangle CAD'. Then the area of concern is that of the rectangle with height 5√3 and length 5+4√3. The area is then ...

  Area = lh = (5√3)(5 +4√3) = 5·5√3 +5·4·3

  Area = 60 +25√3 . . . square units

_____

In the figure, 6.93 = 4√3, and 8.66 = 5√3, 16.93 = 10+4√3.

What is the solution for z?
24=z/0.6?

Answers

Answer:

[tex]z=14.4[/tex]

Step-by-step explanation:

One is given the following equation:

[tex]24=\frac{z}{0.6}[/tex]

Use inverse operations to solve this equation. Multiply both sides of the equation by (0.6) to undo the division of (0.6).

[tex]24=\frac{z}{0.6}[/tex]

[tex](0.6)(24)=z[/tex]

Simplify,

[tex](0.6)(24)=z[/tex]

[tex]z=14.4[/tex]

An aquarium dimensions are 3 1/4 ft x 2 ft x 1 ft. What is the volume of the aquarium?

Answers

Answer:

6 1/2 ft^3 or 13/2 ft^3 or 6.5 ft^3

Step-by-step explanation:

Use the formula for volume of a rectangular prism (lwh)

(3 1/4)(2) = 6 1/2

(6 1/2)(1) = 6 1/2 ft^3

Water flows through a pipe at a rate of 4 quarts per day. Express this rate of flow in liters per week. Round your answer to the nearest tenth.

Answers

Answer:

26.5 liters

Step-by-step explanation:

We know that 1 gallon is 4 quarts, and 1 gallon is approx. 3.78 liters. There's 7 days in a week, so 7 gallons of water is being flowed through. Multiplying 3.78  by 7, we get 26.46 liters per week. Rounding to the nearest tenth, and we get 26.5 liters.

The following equation is often referred to as Euler's Formula; e^pii+1=0 Use what you know about complex numbers to show that this equation is true. In other words, show that e^pii+1=0 __ If someone could please help me understand the proof and the answer to this ill give you brainliest!! thank you

Answers

Answer:

[tex]\large \boxed{e^{i\pi}+1=0}[/tex]

Step-by-step explanation:

Hello, please consider the following.

For any x real number,

[tex]e^{ix}=cos(x)+i\cdot sin(x)\text{, right? So}\\\\e^{i\pi}=cos(\pi)+i\cdot sin(\pi)\\\\e^{i\pi}=-1+i\cdot 0=-1\\\\\text{ We add 1 to both sides of the equation.}\\\\\large \boxed{e^{i\pi}+1=0}\\[/tex]

Hope this helps.

Do not hesitate if you need further explanation.

Thank you

Find the missing probability. P(A)=15,P(A∪B)=1225,P(A∩B)=7100 ,P(B)=?

Answers

Answer:

p(B) = 8310

Step-by-step explanation:

We will use the addition rule of probability of two events to solve the question. According to the rule given two events A and B;

p(A∪B) = p(A)+p(B) - p(A∩B) where;

A∪B is the union of the two sets A and B

A∩B is the intersection between two sets A and B

Given parameters

P(A)=15

P(A∪B)=1225

P(A∩B)=7100

Required

Probability of event B i.e P(B)

Using the expression above to calculate p(B), we will have;

p(A∪B) = p(A)+p(B) - p(A∩B)

1225 = 15+p(B)-7100

p(B) = 1225-15+7100

p(B) = 8310

Hence the missing probability p(B) is 8310.

For what value(s) of k will the function y=6x^2-8x+k have: a) one zero b) two zeros c) no zeros *this is not multiple choice*

Answers

Answer:

Step-by-step explanation:

Hello, please consider the following.

[tex]6x^2-8x+k=0\\\\\text{We compute the discriminant.}\\\\\Delta = b^2-4ac=8^2-4*6*k=8*8-8*3*k=8*(8-3k)[/tex]

And the we know that if the discriminant is

***** [tex]\Delta[/tex] < 0, meaning 8-3k<0, meaning

[tex]\boxed{k>\dfrac{8}{3}}[/tex]

then, there is no real solution.

***** [tex]\Delta = 0[/tex], meaning

[tex]\boxed{k=\dfrac{8}{3}}[/tex]

There is 1 solution.

***** [tex]\Delta[/tex] > 0, meaning

[tex]\boxed{k<\dfrac{8}{3}}[/tex]

There are 2 solutions.

Thank you

PS: To give more details...

[tex]8-3k=0\\\\\text{Add 3k}\\\\8=3k\\\\\text{Divide by 3}\\\\k=\dfrac{8}{3}[/tex]

4 3/4 - 2 3/8 thanks

Answers

Answer:

2 3/8

Step-by-step explanation:

First get the LCM (Least Common Multiple) for the denominators so we can subtract them.  The denominators would both be 8 because it is the lowest number that they can have to be able to subtract.  (The LCM).  So is 4 3/4 has a denominator of 8 it'll become:

4 3/4= 4 6/8 because if 4 turned into 8, that means it multiplied by 2.  So, you need to multiply 3 by 2 as well which equals 6.  This is it as a ratio:

3:4 into x:8    

So 4 times 2, which means you need to do 3 times 2 as well.  It equals 6.

So now that we know 4 3/4 is 4 6/8, it is time to subtract.  2 3/8 already has a denominator of 8 so we don't need to do anything with that.  Subtract.

4 6/8-2 3/8= 2 3/8.  4-2=2, so the whole number is 2.  6-3= 3, so the numerator (the top number) is 3.  The denominator (the bottom number) always stays the same so it is the same, which is 8.

The distance between two points on a number line can be found by taking the ____A_____ of the ____B_____ of the coordinates. Fill in the blanks in the previous sentence by matching the letter for each blank with the correct answer.

Answers

Answer:

The distance between two points on a number line can be found by taking the absolute value of the difference of the coordinates.

Step-by-step explanation:

Every point can be paired with a number on a number lineThe coordinate is the number associated with a pointTo find the distance between point A and B you subtract the coordinates in any order you like and take the absolute value.

17. PLEASE HELP ME
The area of a playground is 160 yd2. The width of the playground is 6 yd longer than its length. Find the length and width of the playground.

A. length = 16 yd, width = 22 yd

B. length = 16 yd, width = 10 yd

C. length = 22 yd, width = 16 yd

D. length = 10 yd, width = 16 yd

Answers

Answer:

b

Step-by-step explanation:

16x10=160

Your width should be 6yd longer so look at the options and see in which options the width is 6 yd longer. It is longer in option A and D, so this eliminates option B and C. Now that we know it’s not b or c, the width times the length should be 160. For option A the area is 352 not 160, so it’s not option A meaning it has to be option D. Just to check 10 x 16 is 160. So yes option d is your final answer. OPTION D. Hope this helps! Would really appreciate it I’ve given brainliest.
Other Questions
Someone help pleasee Cash dividends of $45,000 were declared during the year. Cash dividends payable were $10,000 at the beginning of the year and $15,000 at the end of the year. The amount of cash for the payment of dividends during the year is please answer the question what's the difference between fetch for me water and fetch water for me Which artist had the greatest influence over the ideas used by the FrenchRoyal Academy?O A. MichelangeloO B. RembrandtO C. PoussinO D. Rubens The slowdown in learning new information has been linked to changes in ________, where individuals manipulate and assemble information when making decisions, solving problems, and comprehending written and spoken language. The opposable thumb is a trait that connects us to other primates; allowing us to grab and use tools and increasing chances for survival. This means it is a(n) __________. A landlord rented an apartment to a person with a wheelchair. The landlord allowed the tenant to install bath rails and replace the bathroom sink vanity with a pedestal sink with lever faucet handles; however, the landlord required the tenant to sign a restoration agreement that the accommodations would be restored to their former condition at the end of the lease. In addition, the landlord required the tenant to pay sufficient funds (over a period of time) into an escrow account to cover the bathroom restoration. Has the landlord violated the Fair Housing Act Find the value of x to the nearest tenth. Read the following excerpt from Act I, Scene 1 of Lorraine Hansberry's A Raisin in the Sun:RUTH (With a frown) Bobo? WALTER Yeah. You see, this little liquor store we got in mind cost seventy-five thousand and we figured the initial investment on the place be 'bout thirty thousand, see. That be ten thousand each. Course, there's a couple of hundred you got to pay so's you don't spend your life just waiting for them clowns to let your license get approved RUTH You mean graft? WALTER (Frowning impatiently) Don't call it that. See there, that just goes to show you what women understand about the world. Baby, don't nothing happen for you in this world 'less you pay somebody off! RUTH Walter, leave me alone! (She raises her head and stares at him vigorously then says, more quietly) Eat your eggs, they gonna be cold. WALTER (Straightening up from her and looking off) That's it. There you are. Man say to his woman: I got me a dream. His woman say: Eat your eggs. (Sadly, but gaining in power) Man say: I got to take hold of this here world, baby! And a woman will say: Eat your eggs and go to work. (Passionately now) Man say: I got to change my life, I'm choking to death, baby! And his woman say (In utter anguish as he brings his fists down on his thighs) Your eggs is getting cold! RUTH (Softly) Walter, that ain't none of our money. WALTER (Not listening at all or even looking at her) This morning, I was lookin' in the mirror and thinking about it . . . I'm thirty-five years old; I been married eleven years and I got a boy who sleeps in the living room (Very, very quietly) and all I got to give him is stories about how rich white people live . . . RUTH Eat your eggs, Walter. WALTER (Slams the table and jumps up)Identify two themes that are clearly developed in this excerpt. How does the author develop the two themes over the course of this excerpt? Be sure to use specific details from the text to support your answer. Your baseball team has won 6 games and lost 4 games. If the teamdoes not lose any more games, how many games must the team winto have a win : loss ratio of 2:1? Explain your answer. Put the vocabulary words in order from largest level of organization to smallest. Reorder answers 1. Population Reorder answers 2. Organism Reorder answers 3. Organ Reorder answers 4. Atom Reorder answers 5. Biosphere Reorder answers 6. Species Reorder answers 7. Organ system Reorder answers 8. Community Reorder answers 9. Cell Reorder answers 10. Ecosystem Reorder answers 11. Organelle Reorder answers 12. Tissue Reorder answers 13. Molecule The marked price of a mobile is 25% above its cost price, When it is sold at a gain of10%, the profit amounts to Rs 725. Find the marked price of the mobile. Find the slope of (3,-20), (5,8) Mr. A runs 500 meters in 1 minute, Mr. R runs 450 meters in 50 seconds, Mr. M runs 600 meters in 70 seconds and Mr. K runs 400 meters in 45 seconds. Based on these times, which runner would win a 100 meter race? Gi Gi's Dance Studio provided $280 of dance instruction and rented out its dance studio to the same client for another $165. The client paid immediately. Identify the general journal entry below that Gi Gi's will make to record the transaction. What are the solutions of x2 + 20 = 12x. 1Two radio stations are playing this week's #1 hit song. One radio station plays the song every 18 minutes. The other radio station plays the song every 24 minutes. Both stations play the song at 3pm. How many minutes will pass before they play the same song again. HELP ME PLZZZ!! ILL MARK YOU BRAINLIEST Which statement is an ineffective claim? Fast food restaurants are dangerous to America's youth. The driving age should be raised for three clear reasons. The United States should increase spending for nuclear power. Violent video games make teenagers more violent. The principle that each World Trade Organization member must accord to all other member countries tariff treatment no less favorable than it provides to any other country is known as the __________ principle.